An increasing function f : N ≥ 0 → N ≥ 0 satisfies f ( 2 ) = 7 and
f ( m n ) = f ( m ) + f ( n ) + f ( m ) f ( n ) for all m , n ∈ N ≥ 0 .
Find the remainder when f ( 2 0 1 7 ) is divided by 2 0 1 7 .
This section requires Javascript.
You are seeing this because something didn't load right. We suggest you, (a) try
refreshing the page, (b) enabling javascript if it is disabled on your browser and,
finally, (c)
loading the
non-javascript version of this page
. We're sorry about the hassle.
Observe that f ( 0 ) = f ( 1 ) = 0 . For n ≥ 2 , let us define g ( n ) = f ( n ) + 1 . Then g ( 2 ) = 8 and
g ( m n ) = f ( m n ) + 1 = f ( m ) + f ( n ) + f ( m n ) + 1 = ( f ( m ) + 1 ) ( f ( n ) + 1 ) = g ( m ) g ( n )
Fix an integer n > 2 and consider a sequence p k / q k , k ≥ 1 , of rational numbers that are greater than lo g 2 n and converge to lo g 2 n ( here p k and q k are integers). Then from n < 2 p k / q k , so by monotonicity of g ,
g ( n q k ) ≤ g ( 2 p k ) .
The multiplicativity of g implies that
g ( n ) ≤ g ( 2 ) p k / q k = 2 3 p k / q k = ( ( 2 p k / q k ) 3 ) .
Passing to the limit with k → ∞ , we find that g ( n ) ≤ n 3 . A similar argument shows that g ( n ) ≥ n 3 . Hence g ( n ) = n 3 , and so f ( n ) = n 3 − 1 for all n is the unique solution to the functional equation.
f ( 2 0 1 7 ) ≡ 2 0 1 6 ( m o d 2 0 1 7 ) . So the answer is 2 0 1 6 .
On what set is this function?
Log in to reply
I have written the solution. Please check it.
How do you identify the function???
@Calvin Lin , sir please change the answer to 1 6 .
From continuation of Priyanshu's solution.
here g ( m n ) = g ( m ) g ( n )
From the structure of the function let's assume that g ( x ) = x k where k is an rational number.
So, putting this into the equation we get both side equal
From given information g ( 2 ) = 8 = 2 3
So the value of k = 3
Log in to reply
That claim that " g ( x ) = x k is not true. An example is that g ( 2 k ) = 2 k but g ( n ) = 0 otherwise.
The solution demonstrates how to use the "increasing function" condition to arrive at the value of g ( 2 0 1 7 ) .
@Priyanshu Mishra ;In your given functional equation, put m=1 and n=2.See what you get the value of f(1).
Log in to reply
I am getting the right answer.
f ( n ) = n 3 − 1 for n = 2 we get f ( 2 ) = 8 − 1 = 7 which is given in question.
@Calvin Lin , please change the answer to 1 6 . And mark the report as resolved. My question is correct.
Log in to reply
Shouldn't the answer be 2016?
Log in to reply
Yes SIR.
It must be 2016. I am so stupid. I did calculation mistake twice!.
Please change the answer.
If m=0 then isn't f(0)=-1?
@Calvin Lin, @Shaun Leong, @Kushal Bose
For your kind information, Prof. Titu Andreescu proposed this problem for IMO 1992. I have taken from their only.
The original question was:
**Find all increasing functions f : N ≥ 0 → N ≥ 0 satisfisfying f ( 2 ) = 7 and
f ( m n ) = f ( m ) + f ( n ) + f ( m ) f ( n ) for all m , n ∈ N ≥ 0 . **.
The solution to this problem showed that there is only one function f ( n ) = n 3 − 1 satisfying the conditions.
What i did was a simple modification. I just asked to find remainder when f ( 2 0 1 7 is divided by 2017.
I don't know what is wrong here. Everyone is reporting the problem. I have posted the solution, then also problem prevails. If no one knows the answer, then it does not directly mean that the problem must be incorrect.
So, kindly tell me that should i delete this problem such that you all and i will be free ? Because this is wasting a lot of time.
Log in to reply
The correct answer should have been 2016, and not 16.
I have resolved those reports now.
Problem Loading...
Note Loading...
Set Loading...
The solution given by priyanshu mishra is not the most elegant solution. since we have f(mn)+1 = (f(m)+1)(f(n)+1), Let me denote statement S that states: "f(x)+1 is divisible by x for all integers x." Now let us assume statement S true. It implies that f(m)+1 is divisible by m and f(n)+1 is divisible by n. so (f(m)+1)(f(n)+1) is divisible by mn. Since this is f(mn)+1 , we also have f(mn)+1 is divisible by mn. Hence statement S is true. Now, since f(2017) +1 is divisible by 2017, we have f(2017) is congruent to 2016 modulo 2017. Hence 2016 is the remainder